3
$\begingroup$

Suppose $f = \left( f_1, f_2, \ldots, f_{n-1} \right) : \mathbb{R}^{n} \mapsto \mathbb{R}^{n-1}$ is a $C^{2}$ function, then show that the symbolic determinant

\begin{align} \begin{vmatrix} \frac{\partial}{\partial x_{1}} &\frac{\partial f_{1}}{\partial x_{1}} &\frac{\partial f_{2}}{\partial x_{1}} &\cdots &\frac{\partial f_{n-1}}{\partial x_{1}} \\\\ \frac{\partial}{\partial x_{2}} &\frac{\partial f_{1}}{\partial x_{2}} &\frac{\partial f_{2}}{\partial x_{2}} &\cdots &\frac{\partial f_{n-1}}{\partial x_{2}} \\ \vdots &\vdots &\vdots &\ddots &\vdots \\ \frac{\partial}{\partial x_{n}} &\frac{\partial f_{1}}{\partial x_{n}} &\frac{\partial f_{2}}{\partial x_{n}} &\cdots &\frac{\partial f_{n-1}}{\partial x_{n}} \end{vmatrix} \end{align}

vanishes identically.

I have been trying to rack my brains thinking of various methods which can be used to solve the following problem, but I am getting nowhere, I am not particularly good at theoretical multivariable calculus, and hence might be missing some basic concept here. I would be thankful if someone could point out a direction for me to work through.

P.S. This problem is from the entrance examination, 2010 to the Graduate School at Chennai Mathematical Institute.

1 Answers 1

3

Let $D$ denote the determinant in question. Observe: $D = \sum_I \epsilon_{i_1i_2\dots i_n}\partial_{i_1}\partial_{i_2}f_1 \cdots \partial_{i_n}f_{n-1}$ But, we sum over antisymmetric $i_1i_2\ldots i_n$ in the antisymmetric symbol against the symmetric $i_1i_2\ldots i_n$ appearing in the derivatives (mixed partials commute given the supposed differentiability). Thus, it is zero.

  • 1
    there is an $n$-ary cross-product on $\mathbb{R}^n$ given by a determinant. I suppose you could view the formula in that light. One way or the other you'll need either to use an argument like the one I gave or a property of determinants. But, be careful because one column is an operator whereas the others are functions. It's a hybrid expression so properties may not apply as you expect. Analogy: $A \cdot B = B \cdot A$ however $\nabla \cdot F \neq F \cdot \nabla$ (lhs is function, rhs is operator)2012-10-26